Xem bài viết đơn
Old 05-01-2014, 11:10 AM   #12
kien10a1
+Thành Viên+
 
kien10a1's Avatar
 
Tham gia ngày: Feb 2011
Đến từ: Vĩnh Yên- Vĩnh Phúc
Bài gởi: 371
Thanks: 43
Thanked 263 Times in 153 Posts
Gửi tin nhắn qua Yahoo chát tới kien10a1
Trích:
Nguyên văn bởi namdung View Post

Tôi có cảm nhận rằng đa thức P(x) là bất khả quy, tuy nhiên cần thời gian để kiểm tra chắc chắn.
Em thấy như thầy Nam Dũng phân tích, thì ta chắc chắn có $\left | Q(1)Q(6) \right |=13 $
Vậy thì mọi chuyện xong rồi chứ nhỉ, chú ý thêm $ Q(1)-Q(6) \vdots 5 $ thì ta suy ra ngay không có đa thức Q hệ số nguyên thỏa mãn.
[RIGHT][I][B]Nguồn: MathScope.ORG[/B][/I][/RIGHT]
 
__________________
Quay về với nơi bắt đầu
kien10a1 is offline   Trả Lời Với Trích Dẫn
 
[page compression: 8.46 k/9.55 k (11.48%)]